Difference between revisions of "2021 Fall AMC 10B Problems/Problem 4"

(Since this problem is a duplicate from the AMC 12 Problem, I redirected accordingly.)
(Tag: New redirect)
 
(7 intermediate revisions by 3 users not shown)
Line 1: Line 1:
At noon on a certain day, Minneapolis is <math>N</math> degrees warmer than St. Louis. At <math>4{:}00</math> the temperature in Minneapolis has fallen by <math>5</math> degrees while the temperature in St. Louis has risen by <math>3</math> degrees, at which time the temperatures in the two cities differ by <math>2</math> degrees. What is the product of all possible values of <math>N?</math>
+
#REDIRECT [[2021_Fall_AMC_12B_Problems/Problem_3]]
 
 
<math>(\textbf{A})\: 10\qquad(\textbf{B}) \: 30\qquad(\textbf{C}) \: 60\qquad(\textbf{D}) \: 100\qquad(\textbf{E}) \: 120</math>
 

Latest revision as of 18:46, 3 January 2022